Jump to content

Visitor's Content

There have been 63 items by Visitor (Search limited from 10-06-2020)



Sort by                Order  

#607597 Đề thi và lời giải VMO 2016

Posted by Visitor on 06-01-2016 - 19:51 in Thi HSG Quốc gia và Quốc tế

Co ban nào có ý tưởng cho cậu 2b ko?

Dễ thấy $lim(b_n-b_{n-1})=0$ mà $b_n$ lại tiến ra +vc nên ta có $[b_n]-[b_{n-1}]=0$ hoặc  $[b_n]-[b_{n-1}]=1$ với mọi $n$

Giả sử cái cần cm sai thì tồn tại $N$ để ${b_i} >= 1/2016$ với mọi $i>N$ . Đặt ${b_i} = {c_i} +1/2016$ thì $0< c_i < 2015/2016$

Do $lim(b_n-b_{n-1})=0$ nên từ chỉ số $m$ nào đó trở đi $b_n-b_{n-1}<1/2016$.

Chọn $k>max(N,m)+1$ và  $[b_k]-[b_{k-1}]=1$ thì có $b_k-b_{k-1}<1/2016$ suy ra $1+c_k - c_{k-1}<1/2016$ hay là $2015/2016+ c_k < c_{k-1} $

Vô lí. Vậy có $đpcm$

 

Cái $Fx$ của diễn đàn này bị lỗi rồi.




#607620 Find all the prime $p$ and natural number $m$ that $...

Posted by Visitor on 06-01-2016 - 20:56 in Số học

Find all the prime $p$ and natural number $m$ that $(p-1)!+1=p^m$

Xét $p > 5$. 

Từ đề bài suy ra $(p-2)! = p^{m-1} + p^{m-2} + .... +1$

Vì $p > 5$ nên $2< (p-1)/2 $ suy ra $ (p-2)! = 2.(p-1)/2. A$ chia hết cho $p-1$. Mà VP lại đồng dư với $m$ $mod$ $p-1$ nên $m$ chia hết cho $p-1$

Suy ra $p^{m} >= p^{p-1} > (p-1)! +1 $. Loại

Vậy $p <= 5$




#607642 CMR: Tồn tại các số tự nhiên a,b,c

Posted by Visitor on 06-01-2016 - 21:54 in Số học

CMR: Tồn tại các số tự nhiên a,b,c thỏa $a^{2}+b^{2}+c^{2}=3abc$ và Min(a,b,c)>2004

Giả sử $a,b,c$ là nghiệm ta sẽ tìm $t$ để cho $a,b,c+t$ cũng là nghiệm tức là $a^2 + b^2 + (c+t)^2 = 3ab(c+t)$ 

Vì $a,b,c$ là nghiệm nên ta đã có  $a^2 + b^2 + c^2 = 3abc$ nên rút ra được $t = 3ab-2c $

Đến đây thấy rằng nên chọn $c$ là số nhỏ nhất ngay từ đầu để đảm bảo $c+t$ dương.

Cứ làm như vậy các bộ nghiệm sẽ tăng dần lên và có $đpcm$ thôi 




#607654 Chứng minh rằng nếu $a,b \in \mathbb{X}$ thì...

Posted by Visitor on 06-01-2016 - 22:15 in Số học

:lol:Đã nhận ra lỗi.
Mình làm như sau: Gọi $a$ là số nguyên dương nhỏ nhất và $b$ là số nguyên âm lớn nhất của $\mathbb{X}$
Trường hợp 1: $a + b > 0 \implies a + b \ge 1 \implies a \ge 1 - b \ge 2$. Mặt khác $a + b \in \mathbb{X}$ và $a + b \in \mathbb{N} < a$ mâu thuẫn với sự nhỏ nhất của $a$.
Trường hợp 2: $a + b < 0$ (tương tự trên)
Trường hợp 3:(Cảm ơn bạn Zaraki đã nhắc nhở mình) $a + b = 0 \implies a = -b$. Khi đó mọi phần tử có dạng $Ma (\forall M \in \mathbb{Z})$ đều sẽ thuộc $\mathbb{X}$. Để ý $0$ cũng là phần tử của $\mathbb{X}$
- Nếu $a = 1$, thì $\mathbb{X} = \mathbb{N}$. Do đó điều cần chứng minh đúng.
- Nếu $a > 1$, gọi $K$ là một phần tử không chia hết cho $a$ của $\mathbb{X}$ (giả sử $K$ dương và có $K > a$ và $1 \le gcd(K; a) < a$). Để ý là khi đó sẽ suy ra tổ hợp tuyến tính $mK + na$ cũng thuộc $\mathbb{X}$. Do đó theo bổ đề Bezout thì $mK + na = gcd(K; a) < a \in \mathbb{X}$, mâu thuẫn với tính nhỏ nhất. Vậy $K$ phải có dạng $Ma$.
Tóm lại $\mathbb{X} = {Ma; M \in \mathbb{Z}}$, do đó có điều cần chứng minh.

Đến đoạn $a>1$ cũng có thể lấy ngay $k$ chia cho $a$ : $k=ma + r$ suy ra $r$ thuộc $X$ do đó $r=a$. Bezout cao siêu quá :)

Tên này có phải Tạ Nguyên ko. Thấy giống quá :)




#607671 $S=\left \{ n \epsilon \mathbb{N}*/M^...

Posted by Visitor on 06-01-2016 - 22:39 in Số học

Cho M là một số nguyên dương $S=\left \{ n \epsilon \mathbb{N}*/M^2\leq n<(M+1)^2 \right \} $. Chứng minh rằng mọi tích $ab$ (a,b thuộc S) phân biệt

Cách khác như sau :)

Cũng giả sử có $ab=cd$ và giả sử rằng $a>b$ , $c>d$ và $a>c$ suy ra $a>c>d>b$

Do $ab=cd$ nên tồn tại các số $x,y,z,t$ để cho $a=xy$, $b=zt$, $c=xt$, $d=yz$ ( đây là bổ đề 4 số :) rất cơ bản, chứng minh rất dễ :) )

Vì $a>c$ nên $y>t$, $a>d$ nên $x>z$ .

Suy ra $ xy \ge (z+1)(t+1) =zt +z+t+1 \ge zt+ 2\sqrt{zt} +1 \ge M^2 + 2M+1 = (M+1)^2 $ Vô lí rồi.

Vậy ta có đpcm

$\ge$




#607680 Chứng minh:a=b=c

Posted by Visitor on 06-01-2016 - 23:20 in Số học

Cho p là số nguyên tố và a,b,c là các số nguyên sao cho:

                                            $a^{n}+pb=b^{n}+pc=c^{n}+pa$ (n nguyên dương)

Chứng minh:a=b=c

Từ giả thiết ta có : $p=\frac{a^n-b^n}{c-b}=\frac{b^n-c^n}{a-c}=\frac{c^n-a^n}{b-a}$

Suy ra $-p^3=\frac{a^n-b^n}{a-b}.\frac{b^n-c^n}{b-c}.\frac{c^n-a^n}{c-a}$

Xét các trường hợp của các thừa số là ra 




#607693 Cho A là một ma trận vuông cấp n có tất cả các phần tử là những số nguyên chẵ...

Posted by Visitor on 07-01-2016 - 01:30 in Đại số tuyến tính, Hình học giải tích

Có ai biết làm thì giúp mình với

Đặt $B$ là ma trận toàn số $0$. $E$ là ma trận đơn vị .

Giả sử có gtr $\lambda$ lẻ.

Ta có : $A \equiv B$ $( mod 2)$, $\lambda E \equiv E$ $(mod 2)$

suy ra $A- \lambda E \equiv E$ $(mod 2)$ nên $det(A-\lambda E) \equiv det(E) \equiv 1$ $(mod 2)$ là lẻ. Vậy vô lí

Ta có đpcm

 




#607762 $\sum \frac{1}{x+y+1} \le 1$

Posted by Visitor on 07-01-2016 - 16:49 in Bất đẳng thức và cực trị

Cho $x,y,z>0$ thỏa $xyz=1$. Chứng minh $\sum \frac{1}{x+y+1} \le 1$

Đặt  $x=a^3$ $y=b^3$ $c=z^3$ suy ra $abc=1$

Áp dụng $a^3 + b^3 \ge ab(a+b) $ và thay $1=abc$ vào dưới mẫu ta có $đpcm$




#607979 Vô số số chính phương dạng $n.2^k-7$

Posted by Visitor on 08-01-2016 - 17:37 in Số học

Cho $k \in Z^{+}$ . Chứng minh tồn tại vô số số chính phượng có dạng $n.2^k-7$ trong đó $n \in Z^{+}$ 

Ta sẽ chứng minh với mỗi $k$ tồn tại số $m_k$ sao cho ${m_k}^2+7 \vdots 2^k$ . Thì $m_k$ chính là số cần tìm.

Ta chứng minh bằng qui nạp. Giả sử bt đúng đến $k$, ta chứng minh nó đúng với $k+1$.

Giả sử $ {m_k}^2+7 = t.2^k$. Chọn $m_{k+1}= m_k + h.2^{k-1}$

Suy ra $ {m_{k+1}}^2+7={m_k}^2+ m_k . h . 2^k +2^{2k-2} +7 = t.2^k + m_k . h . 2^k + 2^{2k-1} = 2^k . (t+ m_k . h) +2^{2k-2}$

Do ${m_k}$ lẻ nên có thể chọn $h$ để $t+ m_k . h$ là chẵn và như vậy ${m_{k+1}}^2+7 \vdots 2^{k+1}$

XOng phần tồn tại. Bây giờ chứng minh vô hạn. Nhưng mà thấy ngay nếu ${m_k}$ thỏa mãn thì ${m_k}+l. 2^k$ cũng thỏa mãn :)) ( $l$ bất kì)

Vậy ta có $đpcm$




#607982 $a-b|a$ và $a-b|b$

Posted by Visitor on 08-01-2016 - 18:07 in Số học

Chứng minh rằng với mỗi số tự nhiên $n$ lớn hơn $1$, luôn tồn tại một tập $S$ gồm $n$ phần tử sao cho với hai phần tử $a,b$ bất kì trong $S$, $a-b|a$ và $a-b|b$ nhưng $a-b$ không là ước của bất kì số nào trong $S$ khác nữa.

Ta chứng minh bằng qui nạp. Giả sử bài toán đúng tới $n$ tức là tồn tại tập $S_{n}=\left \{ a_1,a_2,...a_{n} \right \}$  thỏa đề

Ta xây dựng tập $S_{n+1}$ như sau : Đặt $A=\prod (a_i-a_j)$ với $i,j$ chạy từ $1$ đến $n$

Với mỗi $k$ từ $1$ đến $n$ ta chọn $b_k=Aa_1a_2...a_n+a_k$ . Còn $b_{k+1}=Aa_1a_2...a_n$

Ta thấy $b_i-b_j= a_i-a_j |a_i|b_i$ và $b_i-b_j= a_i-a_j |a_j|b_j$ ; $\forall 1\leq i,j\leq n$

và $b_i-b_j$ ko là ước của $b_k$ nào nữa

Riêng $b_i-b_{n+1}= a_i |gcd(b_i,b_{n+1});\forall 1\leq i\leq n$ và cũng ko là ước của $b_k$ nào nữa.

Vậy tập $S_{n+1}=\left \{ b_1,b_2,...b_{n+1} \right \}$ thỏa đề và ta có $đpcm$




#607989 Chứng minh luôn tồn tại $2$ điểm được nối với cùng số lượng điểm...

Posted by Visitor on 08-01-2016 - 18:37 in Tổ hợp và rời rạc

Cho $2015^{2016}$ điểm phân biệt trên cùng một mặt  phẳng. Một số cặp điểm trong số những điểm này được nối với nhau bằng những đoạn thẳng.Chứng minh  luôn tồn tại $2$ điểm  được nối với cùng số lượng điểm khác.

Bài này như cấp $2$ ý :)) thay $2015^2016=m$. Ta thấy 1 điểm có thể nối với $0,1,2....,m-1$ điểm khác.

+Nếu có $1$ điểm ko nối với điểm nào thì cũng ko có điểm nào nối với cả $m-1$ điểm còn lại suy ra số lượng điểm có thể nối là $2,3,..., m-2$, mà có tận $m-1$ điểm ko FA :)) nên sẽ có $2$ điểm có cùng số lượng điểm nối với chúng

+Nếu ko có điểm nào FA :)) thì số lượng điểm có thể nối là $1,2....,m-1$ mà có tận $m$ điểm nên luôn có $2$ điểm có cùng số điểm nối với chúng.

$đpcm$




#608052 $a-b|a$ và $a-b|b$

Posted by Visitor on 08-01-2016 - 21:17 in Số học

Dạ anh giải thích rõ hơn chỗ này được không ạ? Vì giả sử là ước của một $b_k$ nào đó thì $a_i|a_k$ Từ chỗ này em vẫn chưa tìm ra vô lí.

Ở bước quy nạp $n$ ta đã có $a_i-a_k|a_i$ , kia lại có $a_i|a_k$ nên chỉ có trường hợp duy nhất là $a_k=2a_i$ 

đến đây thì ta quay lại bước quy nạp $n-1$ , với những kí hiệu tương tự trên thì $a_k=2a_i\Leftrightarrow A'.a'_1.a'_2...a'_{n-1}+a'_k=2(A'.a'_1.a'_2...a'_{n-1}+a'_i)$

                                         $\Leftrightarrow A'.a'_1.a'_2...a'_{n-1}=a'_k-2a'_i$

                                       (lưu ý là có thể $a'_k$ hoặc $a'_i$ bằng $0$ )

Hiển nhiên vô lí :))




#611267 $S$ không quá $1$ số lẻ.

Posted by Visitor on 27-01-2016 - 13:10 in Số học

2) Trên mặt phẳng có $2011$ điểm bất kì ,ít nhất $3$ điểm không thẳng hàng. Chứng minh rằng luôn vẽ được một đường tròn qua ba trong số $2011$ đã cho mà $2008$ điểm còn lại không nằm ngoài đường tròn.

Nối 2 điểm ở dưới cùng của tập các điểm đó, gọi là $A$ và $B$ ( tức là tất cả các điểm còn lại sẽ cùng nằm trên 1 nửa mặt phẳng bờ $AB$)

Nối tất cả các điểm còn lại với 2 điểm $A,B$. Ta chọn điểm $C$ mà góc $ACB$ là nhỏ nhất.

Đường tròn ngoại tiếp tam giác $ABC$ sẽ chứa tất cả các điểm còn lại
( chứng minh bằng tứ giác nội tiếp và góc ngoài của tam giác , kết hợp với tính nhỏnhất của góc $ACB$)




#611271 $x^n+y^n+z^n+t^n=u^{n+1}$

Posted by Visitor on 27-01-2016 - 13:29 in Số học

1. Chứng minh rằng phương trình sau có vô số nghiệm nguyên:

$$x^5+y^5+z^5=x^4+y^4+z^4$$

 

Đầu tiên chọn $y=-x$ cho gọn thành $z^5=2x^4+z^4$

chia cả 2 vế cho $z^4$ được $z=2(\frac{x}{z})^4+1$. Dễ quá rồi :)) , chọn ngay $z=2k^4 +1 $ và $x=kz$ là vô số nghiệm luôn :))




#611275 $Cho k \epsilon \mathbb{N} k>1 .Cmr$ ton ta...

Posted by Visitor on 27-01-2016 - 13:55 in Số học

$Cho  k  \epsilon \mathbb{N} , k>1 .Cmr$ ton tai huu han x thoa man:

    $\sigma (x)-x=k$

Cách khác đơn giản hơn nhé :)

Nếu $x$ là số nguyên tố thì vô lí. Xét $x$ là hợp số. Gọi $d$ là một ước của $x$ thì hiển nhiên $\frac{x}{d}$ cũng là ước của $x$ 

Ta chọn $d$ là ước nguyên tố của $x$ mà ko vượt quá $\sqrt{x}$ ( luôn tồn tại )

Suy ra $\sigma (x)\geq 1+x+\frac{x}{d}= 1+x+\frac{x}{d}+d-d\geq1+x+2\sqrt{x}-d\geq 1+x+\sqrt{x}$

kéo theo $\sigma (x)-x\geq 1+\sqrt{x}$.  Do đó sẽ có hữu hạn $x$ thỏa đề.




#611857 $F_{10^{k}}$ $(k{\geq}1)$ luôn tận cùng bằng 5

Posted by Visitor on 30-01-2016 - 22:59 in Số học

Bài 1 (QVLuom) Cho vô hạn quả cân dạng $3^{i}$ với i=0,1,2,... mỗi loại 1 quả. Chứng minh rằng tồn tại một cách đặt duy nhất các quả cân lên chiếc cân đĩa sao cho với một vật có khối lượng k nguyên dương bất kì để cân thăng bằng.

Bài 2 (QVLuom) Chứng minh rằng tồn tại ít nhất 1 số nguyên tố nằm giữa 2 số chính phương liên tiếp $n^{2}$ và $(n+1)^{2}$ với $n{\geq}1$.

Bài 3 (Thelongduong) Chứng minh rằng số $F_{10^{k}}$ $(k{\geq}1)$ luôn tận cùng bằng 5 với $F_{n}$  là số Fibonacci thứ n

Bài 1 : nếu $k$ mà chia $3$ dư $2$ thì đặt quả cân kiểu gì :))
Bài 2 : ta có với $n$ đủ lớn thì ta có $(n+1)^2 < 2n^2 $ . Vậy là :)) nếu bài toán của tác giả là đúng thì nó sẽ mạnh hơn cả bổ đề $Bertrand$ :)) Điều này là ko thể 




#611979 Tìm tất cả các số nguyên tố $p$ sao cho $2^n-2$ không chi...

Posted by Visitor on 31-01-2016 - 18:05 in Số học

Với $p$ là số nguyên tố, đặt $n=\frac{2^{2p}-1}{3}$. Tìm tất cả các số nguyên tố $p$ sao cho $2^n-2$ không chia hết cho $n$

Nếu $p\neq 2,3$ thì $n-1=\frac{4^p-4}{3}=\frac{4(4^{p-1}-1)}{3}\vdots 2p\Rightarrow 2^n-2=2(2^{n-1}-1)\vdots (2^{2p}-1)\vdots n$

Vậy $p$ có thể bằng $2$ hoặc $3$ :v




#611980 Tìm GTNN của: P=$\frac{a^{3}+b^{3}+c^...

Posted by Visitor on 31-01-2016 - 18:12 in Bất đẳng thức - Cực trị

Cho $a,b,c> 0$.Tìm GTNN của: P=$\frac{a^{3}+b^{3}+c^{3}}{abc}+\frac{(a+b+c)^{2}}{a^{2}+b^{2}+c^{2}}$.

@@ 




#612375 $F_{10^{k}}$ $(k{\geq}1)$ luôn tận cùng bằng 5

Posted by Visitor on 01-02-2016 - 23:16 in Số học

thử trường hợp với $k=5$ nhé! ta đặt vật và quả cân $3^{0}$ và quả cân $3^{1}$ lên 1 đĩa đĩa kia đặt quả cân $3^{2}$ là được rồi :)

Ra là mình hiểu nhầm đề bài :) Vậy thì bài này mình quy nạp xem sao :) Giả sử bài toán đúng tới $k-1$ ta sẽ chưng sminh nó đúng với $k$

Bài toán đúng với $t$ thì ta kí hiệu $t-True$ :) . Kí hiệu tập các quả cân làm cho $t-True$ là $A_t$

Nếu $k=3t$ thì theo gtqn $t-True$ , lấy $A_k=3A_t$ là xong , từ đây ta có nhận xét là nếu $k-True$ và $k$ chia hết cho $3$ thì tất cả các quả cân trong $A_k$ đều nặng hơn $1$

Nếu $k=3t+1$ , lấy $A_k$ bằng cách thêm vào $A_{3t}$ quả cân $1$.

Nếu $k=3t+2$ thì ta làm lùi từ $h=3(t+1)$ bằng cách thêm vào đĩa chứa $h$ quả cân $1$ . Do $1$ chưa xuất hiện trong $A_h$ ( suy ra từ nhận xét ở trên)  .

Vậy ta có đpcm




#612379 $F_{10^{k}}$ $(k{\geq}1)$ luôn tận cùng bằng 5

Posted by Visitor on 01-02-2016 - 23:45 in Số học

vậy bạn có cm được bài 2 không? Hoặc co chứng minh được bổ đề đó không? Nhớ là tự làm nhé! Hoặc nếu theo ý kiến của bạn thì bạn có thể chứng minh bài 2 sai không? nếu làm không được thì đừng có nói nhé. Khoa học cần bằng chứng chứ không cần lời nói.

Mình ko đủ khả năng để chứng minh bài toán của bạn đúng hay sai. NHƯNG mình xin kể $1$ câu chuyện như sau :)

Có một định đề mang tên nhà toán học $Bertrand$ là : Với mọi $n>1$ thì giữa $n$ và $2n$ luôn có 1 số nguyên tố. Định đề này sau đó đã được $Chebyshev$ chứng minh.Cm cho định lí này rất khó. Cách sơ cấp nhất mà theo mình nhớ là được đưa lên báo TH và TT năm kia , trong đó dùng tới gần cả chục bổ đề nhỏ.

Như thế là đủ hiểu cm khó như nào.

Quay trở lại bài toán của bạn. Giả sử rằng nó đúng. như ở trên mình có viết là với $n$ đủ lớn thì $(n+1)^2<2n^2$ , tức là với $k$ đủ lớn ( tầm 25 trở đi) thì trong đoạn $[k,2k]$ sẽ chứa đoạn $[n^2,(n+1)^2]$ với $n$ nào đó. Mà theo bài toán của bạn thì đoạn này sẽ chứa ít nhất $1$ số nguyên tố. Do đó định lí $Bertrand$ đã được chứng minh.
Vậy bạn có thể đưa ra chứng minh cho bài toán $2$ để chúng ta có $1$ cách chứng minh rất mới cho định lí $Bertrand$  được ko :)

 

p/s : bạn ạ. diễn đàn là nơi trao đổi thảo luận về các bài toán :) dù mình ko làm được nhưng mình đưa ra ý kiến để thảo luận là đúng với ý chí của diễn đàn rồi. cách nói của bạn như kiểu đây là nơi chỉ để hỏi bài hoặc là đưa bài ra để thách thức vậy :)




#612424 Chứng minh rằng dãy $(p_n)$ bị chặn trên.

Posted by Visitor on 02-02-2016 - 14:16 in Số học

Cho dãy $(p_n)$ là dãy các số nguyên tố thỏa mãn với mọi $n \geqslant 3$ thì ta có $p_n$ là ước nguyên tố lớn nhất của $p_{n-1}+p_{n-2}+2000.$.

Chứng minh rằng dãy $(p_n)$ bị chặn trên. 

Nếu cả $2$ số $p_{n-1}, p_{n-2}$ đều lẻ thì ta có $p_{n-1}+p_{n-2}+2000$ là chẵn. Và do tính lớn nhất của $p_n$ nên suy ra $p_n \leq \frac{p_{n-1}+ p_{n-2}+2000}{2} \leq max(p_{n-1}, p_{n-2}) +1000$

Nếu trong $2$ số $p_{n-1}, p_{n-2}$ có $1$ số bằng $2$ thì $p_n  \leq max(p_{n-1}, p_{n-2}) +2+2000\leq max(p_{n-1}, p_{n-2})+2002$

Từ $2$ trường hợp trên ta rút ra $p_n  \leq max(p_{n-1}, p_{n-2})+2002$ với mọi $n$

Do đó nếu đặt dãy $q_n=max(p_{n},p_{n-1})$ thì ta có $q_n \leq q_{n-1}+2002$ với mọi $n$ . ta đi chứng minh dãy $q_n$ bị chặn là xong.

Ta sẽ chứng minh dãy bị chặn bởi số $M$ thỏa mãn: $ q_1 \leq M$ và tất cả các số $M+1,M+2,...M+2002$ đều là hợp số ( số $M$ này luôn chọn được theo định lí $CRT$ ) , thật vậy : do $q_1< M\Rightarrow q_2\leq q_1+2002<M+2002$ . Lại có là từ $M+1$ đến $M+2002$ ko có số nào là số nguyên tố. Mà $q_2$ là số nguyên tố nên nhất định là $q_2 \leq M$

Cứ như vậy ta sẽ chứng minh được $q_n \leq M$ với mọi $n$




#614893 Tồn tại vô hạn

Posted by Visitor on 14-02-2016 - 13:08 in Số học

Cho một số nguyên dương $a$. Chứng minh rằng với mọi $m$, tồn tại vô hạn số nguyên dương $n$ thỏa mãn số ước số của $na^{n} + 1$ thì chia hết cho $m$.

Chọn $p$ là số nguyên tố lẻ mà $v_p(a^{p-1}-1)=1$

để tránh dài dòng mình dùng $v_p$ để diễn tả cho gọn, chứ ko phải là dùng LTE   :))

1.Ta sẽ chứng minh bằng qui nạp theo $k$ rằng tồn tại vô số $n$ để $v_p(na^{n}+1)=k$             $(*)$

Trước tiên là ta nhận xét rằng nếu $v_p(na^{n}+1)=k$ thỏa mãn thì $v_p(ma^{m}+1)$, với $m=n+tp^k(p-1)$ cũng bằng $k$, thật vậy:

Giả sử $na^{n}+1=p^k.A$ 
Ta có $ma^m+1=(n+tp^k(p-1))a^{n+tp^k(p-1)}=na^n(a^{tp^k(p-1)}-1)-(na^n+1)+tp^k.B$
$=na^n(a^{tp^k(p-1)}-1)+p^k(tB-A)$

Do $(A,p)=(B,p)=1$ nên có thể chọn $t$, $(t,p)=1$ để mà $tB-A\not \vdots p$. Mà $v_p(a^{tp^k(p-1)}-1)=k+1$ nên rõ ràng $v_p(ma^m+1)=k$

Vậy $NX$ ở trên là đúng, từ đây suy ra nếu tồn tại $n$ thì sẽ có vô hạn.Do đó ta chỉ cần chứng minh tồn tại

Bắt đầu qui nạp.

+Với $k=1$ chọn $n=p^2-1$ thì $na^n+1=(p^2-1)a^{p^2-1}+1=p^2a^{p^2-1}-(a^{p^2-1}-1)$

do cách chọn thì ta có $v_p(a^{p^2-1}-1)=1$ nên suy ra $v_p(na^n+1)=1$, thỏa mãn

+Giả sử qui nạp đúng đến $k$ , ta cm vs $k+1$ 

Chọn $m=n+tp^{k+1}(p-1)$ thì ta sẽ có $v_p(ma^m+1)=k+1$, chứng minh giống y hệt như chứng minh nhận xét ở trên.

 

2. từ $(*)$ ta suy ra ngay đpcm




#616292 Chứng minh rằng $B$ có thể chọn số $k$ khéo léo để chỉ mộ...

Posted by Visitor on 21-02-2016 - 20:07 in Số học

Cho một tập hữu hạn $S$ gồm các số nguyên dương. Cả hai người A và B đều biết trước các phần tử của $S$. Họ thực hiện trò chơi như sau: A lấy một số $m$ bất kì thuộc $S$ (không để B biết); tiếp theo B chọn một số $k$ bất kì (không nhất thiết thuộc $S$); sau đó A tính toán và cho $B$ biết giá trị của $d(km)$ với $d(n)$ là số các ước số dương của $n$ để B đoán ra số $m$; làm như vậy cho đến khi B đoán đúng. Chứng minh rằng $B$ có thể chọn số $k$ khéo léo để chỉ một lượt chơi là B có thể tìm được số như yêu cầu.

p.s. Đây là đề qgia của Ukraina năm nay. Mình có một lời giải nhưng không nghĩ nó hay cho lắm, đăng lên để anh em tham khảo cho ý kiến.




#616801 Chứng minh n=1

Posted by Visitor on 25-02-2016 - 07:38 in Số học

Cho 3 số nguyên tố p,q,r thỏa mãn $p^{n}+q^{n}=r^{2}$.

Chứng minh n=1

Giả sử $q=2$

Nếu $n$ lẻ thì ta có $p^n+2^n=(p+2)\frac{p^n+2^n}{p+2}= r^2$ mà $r$ nguyên tố suy ra $p+2=\frac{p^n+2^n}{p+2}= r$

Do đó $n=1$ vì nếu $n\geq 3$ thì $\frac{p^n+2^n}{p+2}> p+2$ vô lí.

Nếu $n$ chẵn, $n=2k$ suy ra $(p^k)^2+(2^k)^2=r^2$ .Dễ thấy $p,2,r$ phân biệt nên đây là bộ $Pitago$ nguyên thủy nên tồn tại $m,n$ nguyên tố cùng nhau, khác tính chẵn lẻ sao cho $p^k=2mn, 2^k=m^2-n^2$ hoặc $2^k=2mn, p^k=m^2-n^2$

Vì $p$ nguyên tố lẻ nên cả $2$ trường hợp đều vô lí.

Vậy $n=1$




#616972 chứng minh x thuộc Z

Posted by Visitor on 25-02-2016 - 23:55 in Số học

cho x^3-x thuộc Z,x^4-x thuộc Z chứng minh x thuộc Z

Bài này thật vi diệu :v 

Giả thiết suy ra $\frac{x^4-x}{x^3-x} \in \mathbb{Q}\Leftrightarrow \frac{x^3-1}{x^2-1}\in\mathbb{Q}\Leftrightarrow \frac{x^2+x+1}{x+1}\in \mathbb{Q}\Leftrightarrow \frac{x^2}{x+1}=b\in\mathbb{Q}$

Suy ra $(x^3-x).\frac{x+1}{x^2}= \frac{x^3+x^2-x-1}{x}=c\in\mathbb{Q}$

 đặt $x^3-x=a$ thì $\frac{a+x^2-1}{x}=c\Rightarrow a+x^2-1=cx$

                                                 mà $\frac{x^2}{x+1}= b\Rightarrow x^2=bx+b$

 

Trừ theo vế ta được $a-1=(c-b)x-b$

Dễ thấy $b\neq c$ nên ta sẽ có $x\in\mathbb{Q}$

Ta xét phương trình hệ số nguyên $t^3-t-(x^3-x)=0$ có $x$ là nghiệm hữu tỉ nên nó cũng là nghiệm nguyên.

$đpcm$